Help with a proof involving the span of a subset

  • Thread starter Thread starter paulrb
  • Start date Start date
  • Tags Tags
    Proof Span
Click For Summary
The discussion focuses on proving that span(S) equals span(S U {v}) if and only if v is an element of span(S). To establish this, two main steps are outlined: first, demonstrating that if v is in span(S), then span(S) equals span(S U {v}), and second, showing that if span(S U {v}) equals span(S), then v must be in span(S). The definition of span as the set of all linear combinations of elements is emphasized to support these arguments. The participants clarify that since v is trivially part of span(S U {v}), the proof hinges on these relationships. The conversation concludes with a successful completion of the proof based on this understanding.
paulrb
Messages
18
Reaction score
1

Homework Statement


Let S be a subset of a vector space V, and let v be an element of V. Show that span(S) = span(S U {v}) if and only if v is an element of span(S)


Homework Equations





The Attempt at a Solution



I'm honestly not sure how to get started, I've spent time looking through the text but it hasn't helped.
 
Physics news on Phys.org
You have to do two things.

1. Suppose that v is in span(S), show that span(S)=span(S u {v}).

2. Show that if span(S u {v}) = span(S) then v is in span(S).

Let's look at 2 and break down what you know.

Definition: span(X) is the space of all linear combinations of objects in X.

You need to show v is in span(S).

You know that v is in span(S u {v}) as it is a linear combination of elements of S U {v} trivially.

You know that span(S u {v}) = span(S).

Hence...

Now try 1.
 
Thanks for your help, it allowed me to finish the rest of the proof.
 
Question: A clock's minute hand has length 4 and its hour hand has length 3. What is the distance between the tips at the moment when it is increasing most rapidly?(Putnam Exam Question) Answer: Making assumption that both the hands moves at constant angular velocities, the answer is ## \sqrt{7} .## But don't you think this assumption is somewhat doubtful and wrong?

Similar threads

Replies
15
Views
2K
  • · Replies 3 ·
Replies
3
Views
2K
Replies
1
Views
1K
  • · Replies 2 ·
Replies
2
Views
2K
  • · Replies 6 ·
Replies
6
Views
2K
  • · Replies 5 ·
Replies
5
Views
4K
Replies
4
Views
2K
  • · Replies 4 ·
Replies
4
Views
2K
  • · Replies 5 ·
Replies
5
Views
2K
Replies
34
Views
3K